Finding the Maximum Value of f(x)=x(1-x)^n in [0,1]: A Calculus Problem

  • Thread starter Thread starter help
  • Start date Start date
  • Tags Tags
    Differentiate
Click For Summary
The discussion revolves around finding the maximum value of the function f(x)=x(1-x)^n within the interval [0,1]. Participants emphasize the need to solve f'(x)=0 using the product rule to find critical points, noting that maximum values can occur at these points or at the endpoints of the interval. There's confusion regarding whether the maximum occurs at x=1, with suggestions to plot the function for various values of n to better understand its behavior. The limit of (n+1)an as n approaches infinity is also mentioned, indicating a deeper exploration of the function's properties. Overall, the conversation highlights the importance of correctly applying calculus principles to solve the problem.
help
Messages
14
Reaction score
0
Homework Statement
help with my work
Relevant Equations
derivative and integral
Hello, good afternoon guys. I need a little help from you: D

1)n a positive integer.
f(x)=x(1-x)^n
Solve the equation f'(x)=0 in 0<x<1

this question I asked and found x = 1 / (x + 1) ok

2)An be the maximum value of f(x)=x(1-x)^n in [0,1]
Calculate lim (n+1)an
n tending to infinity

the maximum value would not be making the derivative equal to zero? would it be x = 1? I did not understand
1.jpg
 
Last edited by a moderator:
Physics news on Phys.org
help said:
Homework Statement:: help with my work
Relevant Equations:: derivative and integral

Hello, good afternoon guys. I need a little help from you: D

1)n a positive integer.
f(x)=x(1-x)^n
Solve the equation f'(x)=0 in 0<x<1

this question I asked and found x = 1 / (x + 1) ok

2)An be the maximum value of f(x)=x(1-x)^n in [0,1]
Calculate lim (n+1)an
n tending to infinity

the maximum value would not be making the derivative equal to zero? would it be x = 1? I did not understand View attachment 264068
You did not do the first part correctly.

What is the derivative f'(x)?
 
  • Like
Likes Delta2 and member 587159
Help, you could use the product rule, then set it equal to zero.
 
help said:
Homework Statement:: help with my work
Relevant Equations:: derivative and integral

the maximum value would not be making the derivative equal to zero? would it be x = 1? I did not understand
The maximum value might be attained at a critical point or at the endpoints of the closed interval.

Try plotting the function for a few values of ##n## to get an idea of what you should get.
 
Question: A clock's minute hand has length 4 and its hour hand has length 3. What is the distance between the tips at the moment when it is increasing most rapidly?(Putnam Exam Question) Answer: Making assumption that both the hands moves at constant angular velocities, the answer is ## \sqrt{7} .## But don't you think this assumption is somewhat doubtful and wrong?

Similar threads

  • · Replies 3 ·
Replies
3
Views
907
  • · Replies 6 ·
Replies
6
Views
2K
  • · Replies 4 ·
Replies
4
Views
2K
  • · Replies 1 ·
Replies
1
Views
2K
Replies
7
Views
1K
  • · Replies 7 ·
Replies
7
Views
2K
  • · Replies 6 ·
Replies
6
Views
1K
  • · Replies 6 ·
Replies
6
Views
2K
  • · Replies 2 ·
Replies
2
Views
1K
Replies
1
Views
2K